Question about the Klein Gordon Propagator

  • Context: Graduate 
  • Thread starter Thread starter maverick280857
  • Start date Start date
  • Tags Tags
    Klein Propagator
Click For Summary
SUMMARY

The discussion centers on the Klein-Gordon propagator as presented in Peskin and Schroeder's "An Introduction to Quantum Field Theory." The user struggles with the calculations on pages 29 and 30, specifically regarding the expression for the Green's function and the integration contours. Key points include the necessity of the factor of {-i} in the propagator and the significance of the condition x^0 > y^0 for the equality of certain expressions. The Feynman prescription for the propagator is also highlighted, emphasizing its role in the calculations.

PREREQUISITES
  • Quantum Field Theory (QFT) fundamentals
  • Understanding of the Klein-Gordon equation
  • Feynman prescription for propagators
  • Complex analysis and contour integration
NEXT STEPS
  • Study the derivation of the Klein-Gordon propagator in Peskin and Schroeder
  • Learn about contour integration techniques in complex analysis
  • Explore the role of theta functions in quantum field theory
  • Review the significance of poles and residues in complex integrals
USEFUL FOR

Students and researchers in quantum field theory, particularly those seeking to understand the Klein-Gordon propagator and its mathematical underpinnings. This discussion is beneficial for anyone grappling with the complexities of QFT calculations.

maverick280857
Messages
1,774
Reaction score
5
Hi,

I'm teaching myself QFT. I don't understand some of the calculations described on pages 29 and 30 of Peskin and Schroeder's book on QFT.

...we can write [\phi(x),\phi(y)] = \langle 0|[\phi(x),\phi(y)]|0\rangle. This can be rewritten as a 4 D integral as follows, assuming for now that x^{0} > y^{0}:

\langle 0|[\phi(x),\phi(y)]|0\rangle = \int \frac{d^{3}p}{(2\pi)^3}\left\{\frac{1}{2E_p}e^{-ip\cdot(x-y)}|_{p^0 = E_{p}} + \frac{1}{-2E_p}}e^{-ip\cdot(x-y)}|_{p^0 = -E_{p}}\right\}

The next step is where I have a problem

=_{x^0 > y^0} \int \frac{d^{3}p}{(2\pi)^3}\int \frac{dp^0}{2\pi i}\frac{-1}{p^2-m^2}e^{-ip\cdot(x-y)}

I think I'm making a mathematical mistake somewhere, but I haven't been able to figure it out yet.

Making a digression, the propagator can be written in momentum space as

\Delta_{F}(p) = \frac{1}{2E_{p}}\left[\frac{1}{p^0 - (E_p - i\epsilon)}-\frac{1}{p^0 + (E_p - i\epsilon)}\right]

(Feynman prescription)

Plugging this back into the expression for the Green's function G(x-x') = \int \frac{d^4 p}{(2\pi)^4}e^{-ip\cdot(x-x')}\frac{1}{(p^0)^2 - E_{p}^2} we get

\Delta_{F}(x-x') = \int \frac{d^{3}p}{(2\pi)^3} \frac{1}{2E_{p}}\int \frac{dp^{0}}{2\pi} e^{ip\cdot(x-x')}\left[\frac{1}{p^0 - (E_p - i\epsilon)}-\frac{1}{p^0 + (E_p - i\epsilon)}\right]

This doesn't give me what I want, but I think I am close to it as there is a -i and two theta functions that enter into the final expression if this is integrated.

Specifically,

1. I don't understand how the extra factor of {-i} in the expression in the quote box above (from Peskin & Schroeder) comes in.

2. What is the motivation for the choice of the contour shown on page 30?

3. What is the significance of x^0 > y^0.

Thanks!
 
Last edited:
Physics news on Phys.org
didn't really follow your calcuation, but it seems to me that you are trying to abtain formula 2.60.. and this one contains theta functions...

1. which extra facor?

2. the motivation is having the last line of 2.54 equal to the precedent one... only with that contour you get the two poles that you need (you need two residues!),,,

3. the significance is that the last line of 2.54 and the line before are equal only if x_0>y_0 (otherwise, as explained the last line is zero)
 
Last edited:

Similar threads

  • · Replies 1 ·
Replies
1
Views
2K
  • · Replies 24 ·
Replies
24
Views
3K
  • · Replies 13 ·
Replies
13
Views
3K
  • · Replies 9 ·
Replies
9
Views
4K
  • · Replies 13 ·
Replies
13
Views
4K
  • · Replies 3 ·
Replies
3
Views
2K
  • · Replies 41 ·
2
Replies
41
Views
6K
  • · Replies 3 ·
Replies
3
Views
2K
  • · Replies 9 ·
Replies
9
Views
2K
  • · Replies 1 ·
Replies
1
Views
2K